You are on page 1of 9

2005 ‫ﻳﻮﻧﻴﻮ‬:‫ﺗﺼﺤﻴﺢ اﻹﻣﺘﺤﺎن اﻟﻤﻮﺣﺪ ﻟﻠﺮﻳﺎﺿﻴﺎت‬

Mohamed AIT LHOUSSAIN‫ ﻓﺎس‬- ‫أﻧﺠﺰ اﻟﺘﺼﺤﻴﺢ اﻷﺳﺘﺎذ ﻣﺤﻤﺪ أﻳﺖ اﻟﺤﺴﻴﻦ‬

01‫اﻟﺘﻤﺮﻳﻦ‬
 1 1   1 1
E ‫ ﻋﻨﺼﺮﻳﻦ ﻣﻦ‬Y =  m '+ , m '−  ‫ و‬X =  m + , m −  ‫( ﻟﻴﻜﻦ‬1
 m' m '  m m
( (m , m ') ∈ (\* ) 2 : ‫)ﲝﻴﺚ‬
1  1  1   1  1 
x =   m +   m '+  +  m −   m '−   : ‫ ﲝﻴﺚ‬X *Y = (x , y ) : ‫ﻟﺪﻳﻨﺎ‬
2  m  m '  m  m ' 
1 1  1
=  2mm '+ 2  = mm '+
2 mm '  mm '
1  1  1   1  1 
y =   m +   m '−  +  m −   m '+  : ‫و‬
2  m  m '  m  m ' 
1 1  1
=  2mm '− 2  = mm '−
2 mm '  mm '
X *Y ∈ E : ‫وﻣﻨﻪ‬
E ‫وﻣﻨﻪ * ﻗﺎﻧﻮن ﺗﺮآﻴﺐ داﺧﻠﻲ ﰲ‬
: ‫( أ( ﺣﺴﺐ اﻟﺴﺆال أﻋﻼﻩ وﺑﺎﶈﺎﻓﻈﺔ ﻋﻠﻰ ﻧﻔﺲ اﻟﱰﻣﻴﺰ ﻟﺪﻳﻨﺎ‬2
 1 1 
ϕ (m ) * ϕ (m ') = X *Y =  mm '+ , mm '−  = ϕ (mm ')
 mm ' mm ' 
. (E ,*) ‫ *\( ﳓﻮ‬, ×) ‫ ﺗﺸﺎآﻞ ﻣﻦ‬ϕ ‫وﻣﻨﻪ‬
.‫ ﴰﻮﱄ‬ϕ ‫ ﻓﺈن‬E ‫ﺣﺴﺐ ﺗﻌﺮﻳﻒ اﺠﻤﻟﻤﻮﻋﺔ‬
 1 1  1 1 
ϕ (X ) = ϕ (Y ) ⇒  m + , m −  =  m '+ , m '−  :‫ﻟﺪﻳﻨﺎ‬
 m m  m' m '
 1 1
m + m = m '+ m '
⇒ ⇒ 2m = 2m ' ⇒ m = m '
1
m − = m '− 1
 m m'
(.‫)ﻳﻜﻔﻲ ﲨﻊ ﻃﺮﰲ اﻟﻨﻈﻤﺔ ﻟﻠﺤﺼﻮل ﻋﻠﻰ اﳌﺘﺴﺎوﻳﺔ ﻣﺎ ﻗﺒﻞ اﻷﺧﲑة‬
.‫ ﺗﻘﺎﺑﻞ‬ϕ ‫إذن‬.‫ ﺗﺒﺎﻳﲏ‬ϕ ‫وﻣﻨﻪ‬
ϕ (\* ) = E : ‫ ﴰﻮﱄ ﻓﺈن‬ϕ ‫ب( ﲟﺎ أن‬
.‫( زﻣﺮة ﺗﺒﺎدﻟﻴﺔ‬E ,*) : ϕ ‫ﺑﺎﻟﺘﺸﺎآﻞ‬ ‫ إذن ﺻﻮرﲥﺎ‬.‫ *\( زﻣﺮة ﺗﺒﺎدﻟﻴﺔ‬, ×) ‫و ﻧﻌﻠﻢ أن‬
 1 1 
: ‫ ﻓﺈن ﳑﺎﺛﻠﻪ هﻮ‬X =  m + , m −  ‫و إذا آﺎن‬ ϕ (1) = (2, 0) : ‫ﻋﻨﺼﺮهﺎ اﶈﺎﻳﺪ هﻮ‬
 m m 
1 1 1 
X ' =ϕ   =  + m, − m 
m  m m 
1 1  1 1
y =m− ‫ و‬x =m+ : ‫ﻧﻀﻊ‬. X =  m + , m −  , m > 0 : ‫( أ( ﻟﻴﻜﻦ‬3
m m  m m
1 m 2 − 2m + 1 (m − 1) 2
x −2= m + −2= = ≥ 0 : ‫ﻟﺪﻳﻨﺎ‬
m m m
x ≥ 2
2 2
 1  1 1
 2 : ‫ وﻣﻨﻪ‬x − y =  m +  −  m −  = 4m
2 2
=4
y = x − 4
2
 m  m m

envoyé par M Mohamed AIT LHOUSSAIN http://arabmaths.site.voila.fr


‫و هﺬا ﻳﻌﲏ أن ‪. X ∈ F :‬‬
‫‪x +y‬‬
‫=‪m‬‬ ‫ﻋﻜﺴﻴﺎ ﻟﻴﻜﻦ ‪ X = (x , y ) / x ≥ 2 y 2 = x 2 − 4‬ﻋﻨﺼﺮا ﻣﻦ ‪. F‬ﻧﻀﻊ ‪:‬‬
‫‪2‬‬
‫ﻟﻨﺒﲔ أن ‪. m > 0‬ﻧﻔﱰض اﻟﻌﻜﺲ أي أن ‪. x + y ≤ 0 :‬ﻟﺪﻳﻨﺎ ‪ y 2 = x 2 − 4 :‬وﻣﻨﻪ ‪:‬‬
‫‪ (x − y )(x + y ) = 4 > 0‬وﻣﻨﻪ ‪ x − y ≤ 0 :‬وﻣﻨﻪ ‪(x + y ) + (x − y ) = 2x ≤ 0 :‬‬
‫وهﺬا ﻳﺘﻨﺎﻗﺾ ﻣﻊ ‪. x ≥ 2 :‬إذن ‪. m > 0 :‬‬
‫‪1‬‬ ‫‪(x + y ) + 4‬‬
‫‪2‬‬
‫⇔ ‪m+ =x‬‬ ‫ﻟﺪﻳﻨﺎ ‪= x ⇔ x 2 + y 2 + 2xy + 4 = 2x 2 + 2xy :‬‬
‫‪m‬‬ ‫) ‪2(x + y‬‬
‫اﻟﻌﺒﺎرة اﻷﺧﲑة ﺻﺤﻴﺤﺔ ﻷﻬﻧﺎ ﺗﻌﲏ ‪ . y 2 = x 2 − 4 :‬ﺑﺎﳌﺜﻞ ‪:‬‬
‫‪1‬‬ ‫‪(x + y ) 2 − 4‬‬
‫‪m−‬‬ ‫⇔ ‪=y‬‬ ‫‪= y ⇔ x 2 + y 2 + 2xy − 4 = 2 y 2 + 2xy‬‬
‫‪m‬‬ ‫) ‪2(x + y‬‬
‫واﻟﻌﺒﺎرة اﻷﺧﲑة ﺗﻌﲏ أﻳﻀﺎ ‪y 2 = x 2 − 4 :‬‬
‫ﲠﺬا ﺑﺮهﻨﺎ ﻋﻠﻰ ﻣﺎ ﻃﻠﺐ ﰲ هﺬا اﻟﺴﺆال‪.‬‬
‫ب( ﻟﺪﻳﻨﺎ ∅ ≠ ‪ F‬ﻷن ﻣﻦ أﺟﻞ ‪ m = 1‬ﻣﺜﻼ ﳓﺼﻞ ﻋﻠﻰ ‪. (2, 0) ∈ F :‬‬
‫‪‬‬ ‫‪1‬‬ ‫‪1 ‬‬ ‫‪‬‬ ‫‪1‬‬ ‫‪1‬‬
‫‪Y =  m '+‬‬ ‫‪, m '−‬‬ ‫ﻟﻴﻜﻦ ‪ X =  m + , m −  = ϕ (m ) :‬و )' ‪ = ϕ (m‬‬
‫‪‬‬ ‫'‪m‬‬ ‫‪m '‬‬ ‫‪‬‬ ‫‪m‬‬ ‫‪m‬‬
‫ﻋﻨﺼﺮﻳﻦ ﻣﻦ ‪. F‬ﻟﺪﻳﻨﺎ‪:‬‬
‫‪m‬‬ ‫‪ 1 ‬‬ ‫‪‬‬ ‫‪1 ‬‬ ‫‪m ‬‬
‫)*‪(F ,‬‬ ‫وﻣﻨﻪ‬ ‫ﻓﺈن ‪X *Y ' ∈ F :‬‬ ‫‪. X *Y ' = ϕ (m ) *ϕ ‬وﲟﺎ أن ‪> 0‬‬ ‫‪ =ϕ m‬‬ ‫‪ =ϕ‬‬ ‫‪‬‬
‫'‪m‬‬ ‫‪ m '‬‬ ‫‪ m '‬‬ ‫‪ m '‬‬
‫زﻣﺮة ﺟﺰﺋﻴﺔ ﻟﻠﺰﻣﺮة )*‪. (E ,‬‬

‫اﻟﺘﻤﺮﻳﻦ‪02‬‬

‫اﳉﺰء اﻷول‪:‬‬

‫‪ (1‬إذا آﺎن ]‪ p ≡ 0 [3‬ﻓﺈن ‪ 3 | p‬وﻣﻨﻪ ‪ p = 3‬ﻷن ‪ p‬أوﱄ وهﺬا ﻣﺴﺘﺤﻴﻞ ﺣﺴﺐ اﳌﻌﻄﻴﺎت‪.‬إذن‬
‫‪ p ≡ 1 [3] :‬أو ]‪. p ≡ 2 [3‬وﻣﻨﻪ ]‪ p 2 ≡ 1 [3‬أو ‪. p 2 ≡ 4 ≡ 1 [3] :‬‬
‫إذن ‪. p 2 ≡ 1 [3] :‬‬
‫‪ (2‬أ( ‪ p‬أوﱄ و ‪ p > 2‬إذن ‪ p‬ﻓﺮدي‪ .‬إذن ‪. (∃q ∈ `) p = 2q + 1 :‬‬
‫وﻣﻨﻪ ‪ p − 1 = 2q :‬و ‪. p + 1 = 2q + 2‬وﻣﻨﻪ ‪. p 2 − 1 = ( p − 1)( p + 1) = 4q (q + 1) :‬‬
‫ب( أﺣﺪ اﻟﻌﺪدﻳﻦ ‪ q‬أو ‪ q + 1‬زوﺟﻲ ؛ إذن )‪ 2 | q(q + 1‬وﻣﻨﻪ‪ 8 | 4q(q + 1) :‬وﻣﻨﻪ‪. p 2 ≡ 1 [8] :‬‬
‫‪ (3‬ﺣﺴﺐ ﻣﺎ ﺳﺒﻖ ‪ 3 | p 2 − 1 :‬و ‪ 8 | p 2 − 1‬و ﲟﺎ أن ‪ 3 ∧ 8 = 1 :‬ﻓﺈن ‪:‬‬
‫‪ 24 = 3.8 | p 2 − 1‬أي ‪p 2 ≡ 1 [24] :‬‬

‫اﳉﺰء اﻟﺜﺎﻧﻲ‪:‬‬
‫‪α1‬‬ ‫‪αk‬‬
‫‪. a = p1 ... pk‬ﲟﺎ أن ‪a ∧ 24 = 1‬‬ ‫‪ (1‬ﻧﻌﺘﱪ اﻟﺘﻔﻜﻴﻚ إﱃ ﺟﺬاء ﻣﻦ ﻋﻮاﻣﻞ أوﻟﻴﺔ ﻟﻠﻌﺪد ‪: a‬‬
‫ﻓﺈن اﻷﻋﺪاد اﻷوﻟﻴﺔ ‪ pi‬ﲣﺎﻟﻒ آﻼ ﻣﻦ ‪ 2‬و ‪. 3‬و ﺑﺎﻟﺘﺎﱄ ﺣﺴﺐ اﳉﺰء اﻷول‪:‬‬
‫‪k‬‬ ‫‪k‬‬
‫]‪ (∀i ∈ {1,..., k}) pi2 ≡ 1 [24‬وﻣﻨﻪ ‪a 2 = ∏ ( pi2 )αi ≡∏ (1)αi ≡1[24] :‬‬
‫‪i =1‬‬ ‫‪i =1‬‬

‫‪ a23 ,..., a1 :‬ﲝﻴﺚ ‪:‬‬ ‫‪ (2‬ﻧﻔﱰض وﺟﻮد أﻋﺪاد ﺻﺤﻴﺤﺔ ﻃﺒﻴﻌﻴﺔ‬


‫‪23‬‬
‫‪. ∑ ak2 = 23997‬‬ ‫و‬ ‫‪(∀k ∈ {1,..., 23}) ak ∧ 24 = 1‬‬
‫‪k =1‬‬

‫ﻟﺪﻳﻨﺎ ﺣﺴﺐ اﻟﺴﺆال اﻟﺴﺎﺑﻖ ‪ (∀k ∈ {1,..., 23}) ak 2 ≡ 1[24] :‬وﲟﺎ أن اﳉﻤﻊ ﻣﻨﺴﺠﻢ ﻣﻊ‬

‫‪envoyé par M Mohamed AIT LHOUSSAIN‬‬ ‫‪http://arabmaths.site.voila.fr‬‬


‫‪23‬‬
‫وﻣﻨﻪ ‪ 23 ≡ 23997 [24] :‬أي أن‪:‬‬ ‫‪∑a‬‬ ‫‪2‬‬
‫‪k‬‬ ‫اﳌﻮاﻓﻘﺔ ﺑﱰدﻳﺪ ‪ 24‬ﻓﺈن ‪≡ 23[ 24] :‬‬
‫‪k =1‬‬
‫‪ 4 | 74‬وهﺬا ﻣﺴﺘﺤﻴﻞ ﻷن‪. 74 = 4.18 + 2 :‬إذن ﻻ‬ ‫أي‬ ‫‪ 24 | 23974‬وﻣﻨﻪ ﺑﺎﳋﺼﻮص ‪4 | 23974 :‬‬
‫ﺗﻮﺟﺪ أﻋﺪاد ﲢﻘﻖ ﻣﺎ ﺳﺒﻖ‪.‬‬

‫اﻟﺘﻤﺮﻳﻦ‪:03‬‬

‫اﳉﺰء اﻷول‬
‫‪−2‬‬ ‫‪−‬‬
‫‪2‬‬
‫‪ (1‬أ( ﻟﺪﻳﻨﺎ ‪ f (0) = 0‬و ‪ lim+ f ( x) = lim+ ( x + 2)e = 0‬ﻷن ‪= −∞ :‬‬
‫‪ lim+‬و ‪. lim e x = 0‬إذن ‪f‬‬ ‫‪x‬‬
‫∞‪x →−‬‬ ‫‪x →0 x‬‬ ‫‪x →0‬‬ ‫‪x→0‬‬

‫ﻣﺘﺼﻠﺔ ﰲ اﻟﻨﻘﻄﺔ ‪ 0‬ﻋﻠﻰ اﻟﻴﻤﲔ‪.‬‬


‫‪2‬‬
‫‪X‬‬ ‫)‪f ( x‬‬ ‫‪x+2 x‬‬
‫‪) lim+‬ﻷن ‪ ( lim X = 0 :‬وﻣﻨﻪ ‪ f :‬ﺗﻘﺒﻞ اﻹﺷﺘﻘﺎق‬ ‫‪= lim+‬‬ ‫ب( ﻟﺪﻳﻨﺎ ‪= 0 :‬‬
‫‪x →+∞ e‬‬ ‫‪x →0‬‬ ‫‪x‬‬ ‫‪x →0‬‬ ‫‪2 2x‬‬
‫‪e‬‬
‫ﰲ ‪ 0‬ﻋﻠﻰ اﻟﻴﻤﲔ و ‪f 'd (0) = 0‬‬
‫‪2‬‬
‫‪ x + 2  −x‬‬
‫‪. (∀x > 0) f '( x) =  1 + 2  e‬وﻣﻨﻪ ‪ f ' > 0‬ﻋﻠﻰ [∞‪. ]0, +‬إذن ‪ f‬ﺗﺰاﻳﺪﻳﺔ ﻗﻄﻌﺎ ﻋﻠﻰ‬ ‫ج(‬
‫‪‬‬ ‫‪x ‬‬
‫[∞‪. [0, +‬‬
‫‪−‬‬
‫‪2‬‬
‫‪−2‬‬
‫‪ lim e‬وﻣﻨﻪ ‪. lim f ( x) = +∞ :‬‬ ‫‪x‬‬
‫وﻣﻨﻪ ‪= 1 :‬‬ ‫‪ (2‬أ( ﻟﺪﻳﻨﺎ ‪= 0 :‬‬
‫‪lim‬‬
‫∞‪x→+‬‬ ‫∞‪x →+‬‬ ‫‪x →+∞ x‬‬

‫ب( ﻧﻀﻊ ‪ . (∀t ≥ 0) u (t ) = e− t + t − 1 :‬ﻟﺪﻳﻨﺎ ‪ (∀t ≥ 0) u '(t ) = −e −t + 1 ≥ 0 :‬وﻣﻨﻪ ‪ u‬ﺗﺰاﻳﺪﻳﺔ ﻋﻠﻰ‬


‫‪ \ +‬ﺑﺎﳋﺼﻮص ‪. (∀t ≥ 0) u (t ) ≥ u (0) = 0 :‬‬
‫‪t2‬‬ ‫‪−t‬‬ ‫‪t2‬‬
‫ﻧﻀﻊ ‪ . (∀t ≥ 0) v(t ) = − e − t + 1 = − u (t ) :‬ﻟﺪﻳﻨﺎ‪:‬‬
‫‪2‬‬ ‫‪2‬‬
‫*‬
‫وﻣﻨﻪ ‪ v‬ﺗﺰاﻳﺪﻳﺔ ﻋﻠﻰ ‪. \ +‬ﺑﺎﳋﺼﻮص ‪:‬‬ ‫‪v '(t ) = t − u '(t ) = t + e− t − 1 = u (t ) ≥ 0‬‬
‫‪. (∀t > 0) v(t ) ≥ v(0) = 0‬‬
‫ﻣﻦ اﻟﺪراﺳﺔ اﻟﺴﺎ ﺑﻘﺔ ﻧﺴﺘﻨﺘﺞ اﳌﺘﻔﺎوﺗﺘﲔ اﳌﻄﻠﻮﺑﺘﲔ ﰲ هﺬا اﻟﺴﺆال‪.‬‬
‫ج( ﳝﻜﻦ أن ﻧﻜﺘﺐ ‪:‬‬
‫‪2‬‬ ‫‪2‬‬
‫‪−‬‬ ‫‪−‬‬
‫‪. f ( x) − x = ( x + 2)e‬ﻧﻄﺒﻖ اﻟﺴﺆال اﻟﺴﺎﺑﻖ ﻣﻦ أﺟﻞ‪:‬‬ ‫‪x‬‬
‫‪− x = ( x + 2)(e‬‬ ‫‪x‬‬
‫‪− 1) + 2‬‬
‫‪2‬‬ ‫‪2‬‬
‫‪2‬‬ ‫‪−‬‬ ‫‪2 2‬‬ ‫‪−‬‬ ‫‪2‬‬ ‫‪2‬‬ ‫‪2‬‬
‫‪− ≤ e −1 < 2 −‬‬
‫‪x‬‬
‫ﳓﺼﻞ ﻋﻠﻰ ‪ 0 ≤ e + − 1 < 2 :‬وﻣﻨﻪ ‪:‬‬
‫‪x‬‬
‫=‪t‬‬
‫‪x‬‬ ‫‪x‬‬ ‫‪x‬‬ ‫‪x‬‬ ‫‪x‬‬ ‫‪x‬‬
‫‪2‬‬
‫‪2‬‬ ‫‪−‬‬ ‫‪ 2 2‬‬
‫‪. − ( x + 2) + 2 ≤ ( x + 2)(e x − 1) + 2 <  2 −  ( x + 2) + 2‬‬ ‫وﻣﻨﻪ ‪:‬‬
‫‪x‬‬ ‫‪x‬‬ ‫‪x‬‬
‫‪ 2 2‬‬ ‫‪(2 − 2 x )( x + 2) + 2 x 2 4 − 4 x 4 4‬‬
‫‪ 2‬‬ ‫‪−‬‬ ‫‪‬‬ ‫(‬ ‫‪x‬‬ ‫‪+‬‬ ‫)‪2‬‬ ‫‪+‬‬ ‫‪2‬‬ ‫=‬ ‫=‬ ‫وﲟﺎ أن ‪= 2 − :‬‬
‫‪x‬‬ ‫‪x‬‬ ‫‪x2‬‬ ‫‪x2‬‬ ‫‪x‬‬ ‫‪x‬‬
‫‪2‬‬ ‫‪−2 x − 4 + 2 x‬‬ ‫‪4‬‬
‫= ‪− ( x + 2) + 2‬‬ ‫‪=−‬‬ ‫و ‪:‬‬
‫‪x‬‬ ‫‪x‬‬ ‫‪x‬‬
‫ﻓﺈﻧﻨﺎ ﳓﺼﻞ ﻋﻠﻰ اﳌﺘﻔﺎوﺗﺔ اﳌﺰدوﺟﺔ اﳌﻄﻠﻮﺑﺔ ﰲ هﺬا اﻟﺴﺆال‪.‬‬
‫‪ 4‬‬ ‫‪ 4 2‬‬
‫د( ﲟﺎ أن ‪ lim  −  = lim  2 −  = 0 :‬ﻓﺈن ‪ lim ( f ( x) − x) = 0 :‬وﻣﻨﻪ اﳌﻨﺤﲎ اﳌﻤﺜﻞ‬
‫∞‪x →+‬‬ ‫∞‪x →+‬‬
‫‪ x‬‬ ‫‪x‬‬ ‫∞‪→+‬‬
‫‪x‬‬ ‫‪x‬‬
‫ﻟﻠﺪاﻟﺔ ‪ f‬ﻳﻘﺒﻞ اﳌﺴﺘﻘﻴﻢ ذا اﳌﻌﺎدﻟﺔ ‪ y = x‬ﻣﻘﺎرﺑﺎ ﻣﺎﺋﻼ ﲜﻮار ∞‪. +‬‬

‫‪ (3‬إﻧﺸﺎء اﳌﻨﺤﲎ واﳌﻘﺎرب ‪:‬‬

‫‪envoyé par M Mohamed AIT LHOUSSAIN‬‬ ‫‪http://arabmaths.site.voila.fr‬‬


‫اﳉﺰء اﻟﺜﺎﻧﻲ‬
‫‪ (1‬ﺑﻨﻔﺲ اﻟﻄﺮﻳﻘﺔ اﳌﺴﺘﻌﻤﻠﺔ ﰲ اﳉﺰء اﻷول ﳒﺪ‪:‬‬
‫‪2‬‬
‫)‪f n ( x‬‬ ‫‪‬‬ ‫‪x‬‬ ‫‪1‬‬ ‫‪‬‬
‫وﻣﻨﻪ ‪ f n‬ﺗﻘﺒﻞ اﻹﺷﺘﻘﺎق ﰲ ‪ 0‬ﻋﻠﻰ اﻟﻴﻤﲔ و ‪. f nd (0) = 0‬‬ ‫‪lim‬‬ ‫‪= lim+  +  x2 = 0‬‬
‫‪x →0 +‬‬ ‫‪x‬‬ ‫‪x‬‬ ‫→‬ ‫‪0‬‬ ‫‪2 n x‬‬
‫‪e‬‬
‫‪‬‬ ‫‪2‬‬ ‫‪2 ‬‬ ‫‪−‬‬
‫‪2‬‬
‫‪ (2‬ﻟﺪﻳﻨﺎ ‪(∀x > 0) f n '( x) =  1 + 2  x +   e x > 0 :‬‬
‫‪ x ‬‬ ‫‪n ‬‬
‫وﻣﻨﻪ ‪ f n‬ﺗﺰاﻳﺪﻳﺔ ﻗﻄﻌﺎ ﻋﻠﻰ ‪. \ +‬‬
‫‪2‬‬
‫‪g n ( x) = f n ( x) −‬‬‫‪(3‬أ( ﻧﻌﺮف اﻟﺪاﻟﺔ ‪ g n‬ﻋﻠﻰ اﺠﻤﻟﺎل [∞‪ I =]0, +‬ب ‪:‬‬
‫‪n‬‬
‫‪ g n‬ﻣﺘﺼﻠﺔ وﺗﺰاﻳﺪﻳﺔ ﻗﻄﻌﺎ ﻋﻠﻰ ‪) I‬ﳍﺎ ﻧﻔﺲ اﻟﺘﻐﲑات ﻣﻊ ‪ f n‬ﻷن ‪.( g 'n = f n ' :‬‬
‫إذن ‪ g n‬ﺗﻘﺎﺑﻞ ﻣﻦ ‪ I‬ﳓﻮ ) ‪. g n ( I‬ﻣﻊ ‪ g n ( I ) =] lim g n ( x), lim g n ( x)[ :‬أي أن ‪:‬‬
‫‪x →0 +‬‬ ‫∞‪x→+‬‬

‫‪2‬‬
‫[∞‪ g n ( I ) =] − , +‬ﺑﺎﳋﺼﻮص ) ‪. 0 ∈ g n ( I‬إذن ‪ 0‬ﻳﻘﺒﻞ ﺳﺎﺑﻘﺎ وﺣﻴﺪا ‪ an‬ﺑﺎﻟﺘﻘﺎﺑﻞ ‪. g n‬وﻣﻨﻪ‬
‫‪n‬‬
‫اﳌﻌﺎدﻟﺔ اﳌﻄﻠﻮﺑﺔ ﺗﻘﺒﻞ اﳊﻞ اﻟﻮﺣﻴﺪ ‪ an‬ﰲ اﺠﻤﻟﺎل ‪. I‬‬
‫ب( ﻟﺪﻳﻨﺎ‪:‬‬
‫‪2‬‬ ‫‪2 2‬‬ ‫‪2 ‬‬ ‫‪− ‬‬
‫‪2‬‬
‫‪ f n+1 ( x) −‬ﻟﻜﻞ ‪ x > 0‬وﻟﻜﻞ ‪ n‬ﻣﻦ *` ‪ .‬وﻣﻨﻪ‬ ‫‪− f n ( x) + =  −‬‬ ‫‪ 1 − e  > 0‬‬
‫‪x‬‬
‫‪n +1‬‬ ‫‪n  n n +1‬‬ ‫‪‬‬
‫اﳌﺘﻔﺎوﺗﺔ اﳌﻄﻠﻮﺑﺔ‪.‬‬
‫‪2‬‬ ‫‪2‬‬
‫‪f n+1 (an ) −‬‬ ‫ج( ﻣﻦ اﺟﻞ ‪ x = an‬ﺗﺼﺒﺢ اﳌﺘﻔﺎوﺗﺔ أﻋﻼﻩ ‪> f n (an ) − = 0 :‬‬
‫‪n +1‬‬ ‫‪n‬‬
‫‪2‬‬
‫ﻓﻬﻲ ﺗﺼﺒﺢ ‪f n+1 (an ) − f n+1 (an+1 ) > 0 :‬‬ ‫) ‪= f n+1 (an+1‬‬ ‫وﲟﺎ أن ‪:‬‬
‫‪n +1‬‬

‫‪envoyé par M Mohamed AIT LHOUSSAIN‬‬ ‫‪http://arabmaths.site.voila.fr‬‬


‫وﲟﺎ أﻧﻪ ﺳﺒﻖ اﻟﱪهﺎن ﻋﻠﻰ أن ‪ f n+1‬ﺗﺰاﻳﺪﻳﺔ ﻓﺒﺎﻟﻀﺮورة ‪. an > an+1 :‬‬
‫وﻣﻨﻪ اﳌﺘﺘﺎﻟﻴﺔ ‪ (an ) n>0‬ﺗﻨﺎﻗﺼﻴﺔ ﻗﻄﻌﺎ‪.‬‬
‫وﲟﺎ أﻬﻧﺎ ﻣﺘﺘﺎﻟﻴﺔ ﻣﻮﺟﺒﺔ ﻓﻬﻲ ﻣﺘﻘﺎرﺑﺔ‪.‬ﻧﻀﻊ آﻤﺎ اﺗﻔﻖ ‪. a = lim an :‬‬
‫∞‪n→+‬‬
‫‪2‬‬
‫‪‬‬ ‫‪2  − an 2‬‬ ‫‪2‬‬
‫أي ‪:‬‬ ‫‪a‬‬
‫‪ n‬‬ ‫‪+‬‬ ‫= ) ‪ f n (an‬أي ‪ e = :‬‬ ‫د( ﺣﺴﺐ ﺗﻌﺮﻳﻒ ‪ an‬ﻓﺈﻧﻪ ﻋﻠﻰ اﻟﺘﺘﺎﱄ ‪:‬‬
‫‪‬‬ ‫‪n‬‬ ‫‪n‬‬ ‫‪n‬‬
‫‪2‬‬ ‫‪2‬‬ ‫‪2‬‬
‫‪−‬‬
‫أي ‪. nan = 2e − 2. :‬‬
‫‪ (nan + 2)e = 2‬أي ‪:‬‬
‫‪an‬‬
‫‪nan + 2 = 2e‬‬ ‫‪an‬‬ ‫‪an‬‬

‫ه( ﻧﻔﱰض أن ‪ . a ≠ 0‬ﲟﺎ أن اﳌﺘﺘﺎﻟﻴﺔ ‪ (an ) n>0‬ﻣﻮﺟﺒﺔ ﻓﺈن ‪. a > 0 :‬ﻟﻴﻜﻦ ‪ n‬ﻋﺪدا‬
‫ﺻﺤﻴﺤﺎ ﻃﺒﻴﻌﻴﺎ ﻏﲑ ﻣﻨﻌﺪم‪.‬ﲟﺎ أن اﳌﺘﺘﺎﻟﻴﺔ ‪ (an ) n>0‬ﺗﻨﺎﻗﺼﻴﺔ ﻓﺈن ‪:‬‬
‫أي أن‪:‬‬ ‫‪lim am ≤ lim an = an‬‬
‫∞‪m→+‬‬ ‫∞‪m→+‬‬
‫وﻣﻨﻪ ‪:‬‬ ‫‪( ∀m ≥ n ) am ≤ an‬‬
‫‪1‬‬ ‫‪1‬‬
‫‪1 1‬‬
‫‪( ∀n ≥ 1) e‬‬ ‫‪a‬‬
‫وﻣﻨﻪ ‪≥ e ::‬‬
‫‪an‬‬
‫)‪( ∀n ≥ 1‬‬ ‫≥‬ ‫وﻣﻨﻪ‬ ‫‪( ∀n ≥ 1) a ≤ an‬‬
‫‪a an‬‬
‫‪2‬‬
‫‪2‬‬
‫‪2e − 2‬‬‫‪an‬‬
‫وﻣﻨﻪ ‪:‬‬ ‫= ‪an‬‬ ‫ﻓﺈن ‪:‬‬ ‫‪nan = 2e − 2.‬‬ ‫‪an‬‬
‫ﺣﺴﺐ اﻟﻌﻼﻗﺔ ‪:‬‬
‫‪n‬‬

‫‪2‬‬ ‫‪2‬‬
‫‪2‬‬ ‫‪2e − 2‬‬
‫‪an‬‬
‫‪2e‬‬ ‫‪an‬‬
‫‪+2‬‬ ‫‪2‬‬ ‫‪2‬‬
‫‪2e − 2‬‬‫‪an‬‬
‫‪2e + 2 2e a + 2‬‬
‫‪an‬‬
‫≤ ‪an‬‬ ‫=‬ ‫≤‬ ‫=‬ ‫≤‬
‫‪n‬‬ ‫‪n‬‬ ‫‪n‬‬ ‫‪n‬‬ ‫‪n‬‬

‫‪2‬‬
‫‪2e a + 2‬‬
‫‪ lim an = 0‬أي ‪ a = 0 :‬وهﺬا ﻳﺘﻨﺎﻗﺾ ﻣﻊ ﻣﺎ اﻓﱰﺿﻨﺎﻩ‪.‬‬ ‫‪ lim‬ﻓﺈن ‪:‬‬ ‫وﲟﺎ أن ‪= 0 :‬‬
‫∞‪n→+‬‬ ‫∞‪n→+‬‬ ‫‪n‬‬
‫‪.a = 0‬‬ ‫إذن ‪:‬‬

‫اﳉﺰء اﻟﺜﺎﻟﺚ‪:‬‬
‫ﺗﺰاﻳﺪﻳﺔ ﻗﻄﻌﺎ ﻋﻠﻰ ‪ \ +‬ﻓﺈن‪:‬‬ ‫‪ (1‬أ( ﻟﻜﻞ ‪ x > 0‬ﻟﺪﻳﻨﺎ ‪. x < 2 x :‬ﲟﺎ أن اﻟﺪاﻟﺔ ‪f‬‬
‫وﻣﻨﻪ ‪:‬‬ ‫)]‪( ∀t ∈ [ x, 2 x‬‬ ‫)‪f ( x) ≤ f (t ) ≤ f (2 x‬‬
‫‪2x‬‬ ‫‪2x‬‬ ‫‪2x‬‬
‫‪. xf ( x) ≤ F ( x) ≤ xf (2 x) :‬‬ ‫أي‬ ‫∫‬ ‫‪x‬‬
‫∫ ≤ ‪f ( x) dt‬‬
‫‪x‬‬
‫∫ ≤ ‪f (t ) dt‬‬
‫‪x‬‬
‫‪f ( 2 x) dt‬‬
‫ب( ﻧﻌﻠﻢ أن ‪ lim f ( x) = +∞ :‬وﻣﻨﻪ ‪ lim xf ( x) = +∞ :‬وﺣﺴﺐ اﳌﺘﻔﺎوﺗﺔ اﻷوﱃ أﻋﻼﻩ ﻓﺈن‪:‬‬
‫∞‪x→+‬‬ ‫∞‪x→+‬‬

‫∞‪lim F ( x) = +‬‬
‫∞‪x →+‬‬

‫‪ (2‬أ( ﲟﺎ أن ‪ f‬ﻣﺘﺼﻠﺔ ﻋﻠﻰ ‪ \ +‬ﻓﺈﻬﻧﺎ ﺗﻘﺒﻞ داﻟﺔ أﺻﻠﻴﺔ ‪ G‬ﻋﻠﻰ ‪. \ +‬ﻟﺪﻳﻨﺎ‪:‬‬


‫)‪ . ( ∀x ≥ 0 ) F ( x) = [G ( x) ] x = G (2 x) − G ( x‬وﻣﻨﻪ ‪ F‬ﺗﻘﺒﻞ اﻹﺷﺘﻘﺎق ﻋﻠﻰ ‪) \ +‬ﻷﻬﻧﺎ ﳎﻤﻮع‬
‫‪2x‬‬

‫ﻣﺮآﺒﺎت ﻟﺪوال ﻗﺎﺑﻠﺔ ﻟﻺﺷﺘﻘﺎق(‬


‫ب( ﻣﻦ أ( ﻧﺴﺘﻨﺘﺞ أن ‪( ∀x ≥ 0 ) F '( x) = 2G '(2 x) − G '( x) = 2 f (2 x) − f ( x) :‬‬
‫إذا آﺎن ‪ x = 0‬ﻓﺈن ‪. F '(0) = Fd '(0) = 2 f (0) − f (0) = 0 :‬‬
‫‪−1‬‬ ‫‪−2‬‬
‫إذا آﺎن ‪ x > 0‬ﻓﺈن ‪F '( x) = 2(2 x + 2)e − ( x + 2)e :‬‬
‫‪x‬‬ ‫‪x‬‬

‫‪−2‬‬ ‫‪1‬‬
‫وﳝﻜﻦ أن ﻧﻜﺘﺐ ‪:‬‬ ‫ﻟﺪﻳﻨﺎ ‪F '( x) = e x (2(2 x + 2)e x − ( x + 2)) :‬‬
‫‪1‬‬ ‫‪1‬‬ ‫‪1‬‬ ‫‪1‬‬ ‫‪1‬‬ ‫‪1‬‬ ‫‪1‬‬
‫)‪2(2 x + 2)e − ( x + 2) = 4 xe + 4e − ( x + 2) = xe + 3 xe + 2e + 2e − ( x + 2‬‬
‫‪x‬‬ ‫‪x‬‬ ‫‪x‬‬ ‫‪x‬‬ ‫‪x‬‬ ‫‪x‬‬ ‫‪x‬‬

‫‪envoyé par M Mohamed AIT LHOUSSAIN‬‬ ‫‪http://arabmaths.site.voila.fr‬‬


‫‪1‬‬ ‫‪1‬‬ ‫‪1‬‬ ‫‪1‬‬
‫‪= ( x + 2)e x − ( x + 2) + (3x + 2)e x = ( x + 2)(e x − 1) + (3 x + 2)e x‬‬
‫وﻣﻨﻪ اﳌﺘﺴﺎوﻳﺔ ﺑﺎﻟﺸﻜﻞ اﳌﻄﻠﻮب ﰲ اﻟﺴﺆال‪.‬‬
‫‪ (3‬ﻟﻨﺪرس إﺷﺎرة اﻟﺪاﻟﺔ اﳌﺸﺘﻘﺔ ' ‪F‬‬
‫‪1‬‬
‫‪1‬‬
‫وﻣﻨﻪ ‪ e x − 1 ≥ 0 :‬وﻣﻨﻪ ‪. F '( x) > 0 :‬‬ ‫إذا آﺎن ‪ 0 < x ≤ 1 :‬ﻓﺈن "‪≥ 1 :‬‬
‫‪x‬‬
‫إذا آﺎن ‪ x > 1 :‬ﻓﺈن ‪:‬‬
‫‪1‬‬ ‫‪1‬‬ ‫‪1‬‬ ‫‪1‬‬
‫‪1‬‬ ‫‪1‬‬
‫‪e > 1 − e ⇔ 2e > 1 ⇔ e x > ⇔ > − ln 2‬‬
‫‪x‬‬ ‫‪x‬‬ ‫‪x‬‬
‫‪2‬‬ ‫‪x‬‬
‫وﲟﺎ أن اﻟﻄﺮف اﻷﺧﲑ ﰲ هﺬﻩ اﻟﺘﻜﺎﻓﺆات اﳌﺘﺘﺎﻟﻴﺔ ﺻﺤﻴﺢ ﻓﺈن اﻟﻄﺮف اﻷول ﺻﺤﻴﺢ‬
‫أﻳﻀﺎ‪.‬ﻟﺪﻳﻨﺎ أﻳﻀﺎ ‪3 x + 2 > x + 2 :‬‬
‫ﲟﺎ أن اﻟﻜﻞ ﻣﻮﺟﺐ ﻗﻄﻌﺎ ﻧﺴﺘﻨﺘﺞ ‪:‬‬
‫‪1‬‬ ‫‪1‬‬ ‫‪1‬‬ ‫‪1‬‬
‫)‪ e x (3x + 2) > (1 − e x )( x + 2‬وﻣﻨﻪ ‪ ( x + 2)(e x − 1) + (3 x + 2)e x > 0 :‬وﻣﻨﻪ ‪. F '( x) > 0‬‬
‫وﻣﻨﻪ ‪ F‬ﺗﺰاﻳﺪﻳﺔ ﻗﻄﻌﺎ ﻋﻠﻰ ‪. \ +‬‬
‫ﺟﺪول اﻟﺘﻐﲑات ﻟﻠﺪاﻟﺔ ‪: F‬‬

‫اﻟﺘﻤﺮﻳﻦ‪04‬‬
‫)‪f (iy ) = iy ⇔ i (iy ) − 1 = iy (iy + 1‬‬ ‫‪2‬‬
‫‪(1‬أ(‬
‫) ‪⇔ − y − 1 = iy (− y 2 + 1 + 2iy ) = −2 y 2 + i (− y 3 + y‬‬
‫‪⇔ 2 y 2 − y − 1 + i( y 3 − y) = 0 ⇔ 2 y 2 − y − 1 = ( y 3 − y) = 0‬‬
‫‪⇔ y =1‬‬
‫ﻷن ‪ 0‬و ‪ −1‬ﻻ ﳛﻘﻘﺎن إﻻ اﻟﻌﻼﻗﺔ اﻟﺜﺎﻧﻴﺔ دون اﻷوﱃ‪.‬‬
‫أذن ‪. f (i ) = i‬‬
‫ب( اﳌﻌﺎدﻟﺔ ‪ f ( z ) = z‬ﺗﻜﺎﻓﺊ ‪ z ≠ −i :‬و ‪ z ( z + 1) = iz − 1‬ﺗﻜﺎﻓﺊ ‪:‬‬
‫‪2‬‬

‫‪ z ≠ −i‬و ‪. z 3 + 2 z 2 + (1 − i ) z + 1 = 0‬ﺳﻨﻌﻤﻞ اﳊﺪودﻳﺔ ﻣﻦ اﻟﺪرﺟﺔ اﻟﺜﺎﻟﺜﺔ ﺑﺎﺳﺘﻌﻤﺎل ﻧﺘﻴﺠﺔ‬


‫اﻟﺴﺆال اﻟﺴﺎﺑﻖ ‪:‬‬
‫ﻧﻔﱰض أن ‪. z + 2 z + (1 − i ) z + 1 = ( z − i )( z + az + i ) :‬ﲟﻘﺎرﻧﺔ ﻣﻌﺎﻣﻼت اﳊﺪاﻧﻴﺎت ﻣﻦ اﻟﺪرﺟﺔ‬
‫‪3‬‬ ‫‪2‬‬ ‫‪2‬‬

‫اﻟﺜﺎﻧﻴﺔ ﳒﺪ أن ‪ 2 = a − i :‬وﻣﻨﻪ ‪a = 2 + i :‬‬


‫اﳌﻌﺎدﻟﺔ ﻣﻦ اﻟﺪرﺟﺔ اﻟﺜﺎﻧﻴﺔ ‪ z + (2 + i ) z + i = 0 :‬ﺗﻘﺒﻞ اﳌﻤﻴﺰ ‪:‬‬
‫‪2‬‬

‫‪∆ = (2 + i ) 2 − 4i = 3‬‬
‫‪−2 − 3 − i‬‬ ‫‪−2 + 3 − i‬‬
‫= ‪z2‬‬ ‫و‬ ‫= ‪z1‬‬ ‫وﻣﻨﻪ ﺟﺬرا اﳌﻌﺎدﻟﺔ ‪:‬‬
‫‪2‬‬ ‫‪2‬‬
‫وﻣﻨﻪ ﺟﺬور اﳌﻌﺎدﻟﺔ ‪: f ( z ) = z :‬‬
‫‪−2 − 3 − i‬‬ ‫‪−2 + 3 − i‬‬
‫= ‪z2‬‬ ‫و‬ ‫= ‪z1‬‬ ‫‪ z0 = i‬و‬
‫‪2‬‬ ‫‪2‬‬
‫‪ (2‬أ( ﻟﺪﻳﻨﺎ ‪:‬‬
‫‪7π‬‬ ‫‪π‬‬ ‫‪11π‬‬
‫‪− 3 −i‬‬ ‫‪i‬‬ ‫‪3 −i‬‬ ‫‪−i‬‬ ‫‪i‬‬
‫= ‪z2 + 1‬‬ ‫‪=e 6‬‬ ‫= ‪ z1 + 1‬و‬ ‫‪=e =e 6‬‬
‫‪6‬‬
‫‪2‬‬ ‫‪2‬‬
‫ب( ﻧﺴﺘﻨﺘﺞ ‪:‬‬

‫‪envoyé par M Mohamed AIT LHOUSSAIN‬‬ ‫‪http://arabmaths.site.voila.fr‬‬


11π 11π
i i  − i1112π i
11π
 11π i1112π
z1 = −1 + e 6
=e 12
 − e + e 12
 = 2i sin e
  12
 π 11π   17π 
11π i  2 + 12  11π i  12 
= 2sin e = 2sin e
12 12
 11π 17π 
z1 =  2sin , : ‫وﻣﻨﻪ اﻟﺸﻜﻞ اﳌﺜﻠﺜﻲ‬
 12 12 
: ‫ﺑﺎﳌﺜﻞ وﺑﺎﺗﺒﺎع ﻧﻔﺲ اﻟﻄﺮﻳﻘﺔ‬
7π  π 7π   13π 
7π i  2 + 12  i 7π i  12 
z2 = −1 + e = 2sin e = 2sin 6
e
12 12
 7π 13π 
z2 =  2sin , : ‫أي‬
 12 12 
: ‫ ﺑﺎﺳﺘﻌﻤﺎل ﺷﻜﻠﻬﻤﺎ اﳉﱪي‬z2 ‫ و‬z1 ‫ﳝﻜﻦ ﺣﺴﺎب ﻣﻌﻴﺎر آﻞ ﻣﻦ‬
1 1
: ‫ | وﻣﻨﻪ‬z1 |= 2 − 3. : ‫ | وﻣﻨﻪ‬z1 |2 = (( 3 − 2) 2 + 1) = (8 − 4 3) = 2 − 3
4 4
 17π 
z1 =  2 − 3 ,
 12 
1 1
: ‫ | وﻣﻨﻪ‬z2 |= 2 + 3 : ‫ | وﻣﻨﻪ‬z2 |2 = ((− 3 − 2) 2 + 1) = (8 + 4 3) = 2 + 3
4 4
 13π 
z2 =  2 + 3 ,
 12 
. 0 ≤ α < π ‫ ﲝﻴﺚ‬z = eiα (3
: ‫أ( ﻟﺪﻳﻨﺎ‬
ieiα − 1 −ie−iα − 1
(1) f ( z) = =
( ) ( )
2 2
eiα + 1 e−iα + 1

ieiα − 1 ieiα − 1
izf ( z ) = ieiα = ieiα
( ) ( )
2 2
eiα + 1 ei 2α e−iα + 1

ieiα − 1 −ie−iα − 1
(2) izf ( z ) = ie−iα = : ‫وﻣﻨﻪ‬
( ) ( )
2 2
e−iα + 1 e−iα + 1

f ( z ) = izf ( z ) : ‫( ﻧﺴﺘﻨﺘﺞ أن‬2) ‫( و‬1) ‫ﻣﻦ‬


: ‫ب( ﺣﺴﺐ اﻟﻌﻼﻗﺔ أﻋﻼﻩ ﻟﺪﻳﻨﺎ‬
f ( z ) + f ( z ) = 0 ⇔ f ( z ) + izf ( z ) = 0 ⇔ f ( z )(1 + iz ) = 0
⇔ f ( z) = 0 ‫أو‬ (1 + iz ) = 0
−1
⇔ z = z0 ‫أو‬ z = z1 ‫أو‬ z = z2 ‫أو‬ z= =i
i
‫وﻣﻨﻪ أﻷﻋﺪاد اﻟﻌﻘﺪﻳﺔ اﻟﱵ ﲢﻘﻖ هﺬﻩ اﻟﻌﻼﻗﺔ هﻲ اﳊﻠﻮل اﻟﺜﻼﺛﺔ ﻟﻠﻤﻌﺎدﻟﺔ‬
‫هﻲ‬ ‫ اﳌﻤﻜﻨﺔ هﻲ‬α ‫وﻗﺪ ﺳﺒﻖ أن أﻋﻄﻴﻨﺎ اﻟﺸﻜﻞ اﳌﺜﻠﺜﻲ ﻟﻜﻞ ﻣﻨﻬﺎ وﻣﻨﻪ ﻗﻴﻢ‬. f ( z ) = z
17π 13π π
‫وﻣﻊ اﻟﺸﺮط‬. 2π ‫أو أﻋﺪاد ﺗﻮاﻓﻘﻬﺎ ﺑﱰدﻳﺪ‬ ‫و‬ ‫و‬ : ‫ﻋﻠﻰ اﻟﺘﻮاﱄ‬
12 12 2

envoyé par M Mohamed AIT LHOUSSAIN http://arabmaths.site.voila.fr


π
α= ‫ ﺗﺒﻘﻰ ﻗﻴﻤﺔ وﺣﻴﺪة وهﻲ‬0 ≤ α < π
2
θ
θ i
( ∀θ ∈ \ ) 1 + e iθ
= 2cos e 2 (1) : ‫ج( ﺳﻨﺴﺘﻌﻤﻞ اﻟﺼﻴﻐﺔ اﻵﺗﻴﺔ‬
2
( ‫ اﻟﱪهﺎن ﻋﻠﻴﻬﺎ ﻣﻌﺮوف وﺳﻬﻞ‬:‫)ﺻﻴﻐﺔ ﻧﺼﻒ اﻟﺰاوﻳﺔ‬
 i α +  
 3π 

: ‫ وﻣﻨﻪ ﺣﺴﺐ اﻟﺼﻴﻐﺔ أﻋﻼﻩ‬ieiα − 1 = −(1 − ieiα ) = − 1 + e  2   : ‫ﻟﺪﻳﻨﺎ‬


 
 
 α 3π 
 α 3π  i  2 + 4 
: ‫ﳓﺼﻞ ﻋﻠﻰ‬ −1 = eiπ : ‫وﲟﻼﺣﻈﺔ‬ ieiα − 1 = −2cos  + e
2 4 
 α 3π   α 7π 
i + +π  i + 
iα  α 3π   2 4  = 2cos  α + 3π  e  2 4  (2)
ie − 1 = 2cos  +  e  
2 4  2 4 
: ‫ﻟﺪﻳﻨﺎ أﻳﻀﺎ‬

( )
2  α
α i  α
1 + eiα =  2cos e 2  = 4cos 2 eiα (3)
 2  2
: ‫( ﻧﺴﺘﻨﺘﺞ‬3) ‫( و‬2) ‫ﻣﻦ‬
 α 7π 
i +
 α 3π   2 4 
ieiα − 1
2cos  + e

f ( z ) = f (e ) = =  2 4 
(1 + eiα )
2
α
4cos 2 eiα
2
 α 3π   7π α 
α 2cos  +  i −
2 4   4 2 
i
ie − 1
f ( z) = =  e (4) : ‫أي‬
( )
iα 2 2 α
1+ e 4cos
2
 α 3π
 3π α 3π 5π
: ‫ﻧﻀﻊ‬. cos  +  < 0 : ‫وﻣﻨﻪ‬ ≤ + < : ‫ وﻣﻨﻪ‬0 ≤ α < π : ‫ﻟﺪﻳﻨﺎ‬
2 4 4 2 4 4
  α 3π 
r = − cos  2 + 4 
: ‫ﻟﺪﻳﻨﺎ ﺣﺴﺐ ﻣﺎ ﺳﺒﻖ‬  

ϕ = 7π − α + π ≡ 3π − α [ 2π ]
 4 2 4 2

r > 0 ‫ ﻣﻊ‬f ( z ) = re
α ≠ π ‫ و‬0 ≤ α < 2π : ‫ ﲝﻴﺚ‬z = reiα : ‫(ﳊﻞ اﻟﻨﻈﻤﺔ اﳌﻄﻠﻮﺑﺔ ﻧﻀﻊ‬4
‫( أﻋﻼﻩ ﺗﺒﻘﻰ ﺻﺎﳊﺔ ﻷن اﻟﱪهﺎن ﻋﻠﻴﻬﺎ ﻏﲑ ﻣﺮﺗﺒﻂ ﺑﺎﺠﻤﻟﻤﻮﻋﺔ اﻟﱵ ﻳﻨﺘﻤﻲ‬4) ‫اﻟﺼﻴﻐﺔ‬
. α ‫إﻟﻴﻬﺎ‬
1  α 3π   7π α  α
Re( f ( z )) = ⇒ 2cos  +  cos  −  = 2cos 2 : ‫وﻣﻨﻪ‬
2 2 4   4 2 2
: ‫ﺑﺎﺳﺘﻌﻤﺎل ﺻﻴﻐﺔ اﻟﺘﺤﻮﻳﻞ ﳉﺬاء اﱃ ﳎﻤﻮع ﳓﺼﻞ ﻋﻠﻰ‬
1  5π   α
: ‫أي‬  cos(α − π ) + cos    = cos2
2  2  2
1 α
: ‫ وﻣﻨﻪ‬cosα = − : ‫ أي‬− cos α = 1 + cos α : ‫وﻣﻨﻪ‬ − cos α = 2cos 2
2 2

envoyé par M Mohamed AIT LHOUSSAIN http://arabmaths.site.voila.fr


1 3 1 3 4π 2π
z = − −i = j ‫أو‬ z = − +i = j ‫ وﻣﻨﻪ‬. α = ‫ أو‬α =
2 2 2 2 3 3
: ‫ﻋﻜﺴﻴﺎ ﻧﺘﺤﻘﻖ ﺑﺴﻬﻮﻟﺔ أن اﻟﻌﺪدﻳﻦ اﻟﺴﺎﺑﻘﲔ ﺣﻼن ﻟﻠﻨﻈﻤﺔ وﻣﻨﻪ ﳎﻤﻮﻋﺔ اﳊﻠﻮل‬
S = { j, j }

envoyé par M Mohamed AIT LHOUSSAIN http://arabmaths.site.voila.fr

You might also like